Infimum of the set ${xin mathbb{Q};|;x^2<2}$?












3












$begingroup$


My first year Analysis textbook at university includes examples to grasp the concepts of infimum, supremum, maximum, minimum, lower bound and upper bound in set theory for subsets of the real numbers $mathbb{R}$. One of such examples is the set:
$$S={xin mathbb{Q};|;x^2<2}$$



for which the textbook says that "there does not exist a maximum, the upper bounds are all real numbers greater than or equal to $sqrt{2}$, and the supremum is $sqrt2$".



Now, it also says that for this same subset, "there is no minimum, there are no lower bounds and thus there is no infimum": to me, it would seem like there is no minimum (since one could approach $-sqrt2$ infinitely by smaller and smaller rational numbers part of the set), with the infimum being $-sqrt2$ and lower bounds all real numbers smaller than or equal to $-sqrt2$. Am I wrong, or is the textbook wrong?










share|cite|improve this question











$endgroup$








  • 2




    $begingroup$
    infimum of the set ${xin mathbb{Q};|;x^2<2}$ is $-sqrt {2} $
    $endgroup$
    – Darman
    Dec 23 '18 at 9:38
















3












$begingroup$


My first year Analysis textbook at university includes examples to grasp the concepts of infimum, supremum, maximum, minimum, lower bound and upper bound in set theory for subsets of the real numbers $mathbb{R}$. One of such examples is the set:
$$S={xin mathbb{Q};|;x^2<2}$$



for which the textbook says that "there does not exist a maximum, the upper bounds are all real numbers greater than or equal to $sqrt{2}$, and the supremum is $sqrt2$".



Now, it also says that for this same subset, "there is no minimum, there are no lower bounds and thus there is no infimum": to me, it would seem like there is no minimum (since one could approach $-sqrt2$ infinitely by smaller and smaller rational numbers part of the set), with the infimum being $-sqrt2$ and lower bounds all real numbers smaller than or equal to $-sqrt2$. Am I wrong, or is the textbook wrong?










share|cite|improve this question











$endgroup$








  • 2




    $begingroup$
    infimum of the set ${xin mathbb{Q};|;x^2<2}$ is $-sqrt {2} $
    $endgroup$
    – Darman
    Dec 23 '18 at 9:38














3












3








3





$begingroup$


My first year Analysis textbook at university includes examples to grasp the concepts of infimum, supremum, maximum, minimum, lower bound and upper bound in set theory for subsets of the real numbers $mathbb{R}$. One of such examples is the set:
$$S={xin mathbb{Q};|;x^2<2}$$



for which the textbook says that "there does not exist a maximum, the upper bounds are all real numbers greater than or equal to $sqrt{2}$, and the supremum is $sqrt2$".



Now, it also says that for this same subset, "there is no minimum, there are no lower bounds and thus there is no infimum": to me, it would seem like there is no minimum (since one could approach $-sqrt2$ infinitely by smaller and smaller rational numbers part of the set), with the infimum being $-sqrt2$ and lower bounds all real numbers smaller than or equal to $-sqrt2$. Am I wrong, or is the textbook wrong?










share|cite|improve this question











$endgroup$




My first year Analysis textbook at university includes examples to grasp the concepts of infimum, supremum, maximum, minimum, lower bound and upper bound in set theory for subsets of the real numbers $mathbb{R}$. One of such examples is the set:
$$S={xin mathbb{Q};|;x^2<2}$$



for which the textbook says that "there does not exist a maximum, the upper bounds are all real numbers greater than or equal to $sqrt{2}$, and the supremum is $sqrt2$".



Now, it also says that for this same subset, "there is no minimum, there are no lower bounds and thus there is no infimum": to me, it would seem like there is no minimum (since one could approach $-sqrt2$ infinitely by smaller and smaller rational numbers part of the set), with the infimum being $-sqrt2$ and lower bounds all real numbers smaller than or equal to $-sqrt2$. Am I wrong, or is the textbook wrong?







supremum-and-infimum






share|cite|improve this question















share|cite|improve this question













share|cite|improve this question




share|cite|improve this question








edited Dec 23 '18 at 10:25









Asaf Karagila

305k33436767




305k33436767










asked Dec 23 '18 at 9:31









MewMew

205




205








  • 2




    $begingroup$
    infimum of the set ${xin mathbb{Q};|;x^2<2}$ is $-sqrt {2} $
    $endgroup$
    – Darman
    Dec 23 '18 at 9:38














  • 2




    $begingroup$
    infimum of the set ${xin mathbb{Q};|;x^2<2}$ is $-sqrt {2} $
    $endgroup$
    – Darman
    Dec 23 '18 at 9:38








2




2




$begingroup$
infimum of the set ${xin mathbb{Q};|;x^2<2}$ is $-sqrt {2} $
$endgroup$
– Darman
Dec 23 '18 at 9:38




$begingroup$
infimum of the set ${xin mathbb{Q};|;x^2<2}$ is $-sqrt {2} $
$endgroup$
– Darman
Dec 23 '18 at 9:38










2 Answers
2






active

oldest

votes


















4












$begingroup$

Looking at $S$ as a subset of $mathbb Q$ it has lower bounds, but has no infimum (i.e. greatest lower bound).



Looking at $S$ as a subset of $mathbb R$ it does have an infimum which is $-sqrt2inmathbb R$.






share|cite|improve this answer









$endgroup$













  • $begingroup$
    Would the first case not mean there is also no supremum, though?
    $endgroup$
    – Mew
    Dec 23 '18 at 9:39












  • $begingroup$
    Indeed if $S$ is looked at as a subset of $mathbb Q$ (ordered by $leq$) it has no supremum either. If in the textbook they say that there is a supremum but no infimum then they are not consistent.
    $endgroup$
    – drhab
    Dec 23 '18 at 9:41












  • $begingroup$
    Thus, as it is asserted that there exists a supremum, I may conclude that the textbook contradicts itself and is wrong.
    $endgroup$
    – Mew
    Dec 23 '18 at 9:44










  • $begingroup$
    I think so, yes.
    $endgroup$
    – drhab
    Dec 23 '18 at 9:44



















2












$begingroup$

To me it looks like the textbook is wrong, and you have it right. The set does have lower bounds, and therefore a greatest lower bound (among the reals), and that's $-sqrt2$.






share|cite|improve this answer









$endgroup$













    Your Answer





    StackExchange.ifUsing("editor", function () {
    return StackExchange.using("mathjaxEditing", function () {
    StackExchange.MarkdownEditor.creationCallbacks.add(function (editor, postfix) {
    StackExchange.mathjaxEditing.prepareWmdForMathJax(editor, postfix, [["$", "$"], ["\\(","\\)"]]);
    });
    });
    }, "mathjax-editing");

    StackExchange.ready(function() {
    var channelOptions = {
    tags: "".split(" "),
    id: "69"
    };
    initTagRenderer("".split(" "), "".split(" "), channelOptions);

    StackExchange.using("externalEditor", function() {
    // Have to fire editor after snippets, if snippets enabled
    if (StackExchange.settings.snippets.snippetsEnabled) {
    StackExchange.using("snippets", function() {
    createEditor();
    });
    }
    else {
    createEditor();
    }
    });

    function createEditor() {
    StackExchange.prepareEditor({
    heartbeatType: 'answer',
    autoActivateHeartbeat: false,
    convertImagesToLinks: true,
    noModals: true,
    showLowRepImageUploadWarning: true,
    reputationToPostImages: 10,
    bindNavPrevention: true,
    postfix: "",
    imageUploader: {
    brandingHtml: "Powered by u003ca class="icon-imgur-white" href="https://imgur.com/"u003eu003c/au003e",
    contentPolicyHtml: "User contributions licensed under u003ca href="https://creativecommons.org/licenses/by-sa/3.0/"u003ecc by-sa 3.0 with attribution requiredu003c/au003e u003ca href="https://stackoverflow.com/legal/content-policy"u003e(content policy)u003c/au003e",
    allowUrls: true
    },
    noCode: true, onDemand: true,
    discardSelector: ".discard-answer"
    ,immediatelyShowMarkdownHelp:true
    });


    }
    });














    draft saved

    draft discarded


















    StackExchange.ready(
    function () {
    StackExchange.openid.initPostLogin('.new-post-login', 'https%3a%2f%2fmath.stackexchange.com%2fquestions%2f3050189%2finfimum-of-the-set-x-in-mathbbq-x22%23new-answer', 'question_page');
    }
    );

    Post as a guest















    Required, but never shown

























    2 Answers
    2






    active

    oldest

    votes








    2 Answers
    2






    active

    oldest

    votes









    active

    oldest

    votes






    active

    oldest

    votes









    4












    $begingroup$

    Looking at $S$ as a subset of $mathbb Q$ it has lower bounds, but has no infimum (i.e. greatest lower bound).



    Looking at $S$ as a subset of $mathbb R$ it does have an infimum which is $-sqrt2inmathbb R$.






    share|cite|improve this answer









    $endgroup$













    • $begingroup$
      Would the first case not mean there is also no supremum, though?
      $endgroup$
      – Mew
      Dec 23 '18 at 9:39












    • $begingroup$
      Indeed if $S$ is looked at as a subset of $mathbb Q$ (ordered by $leq$) it has no supremum either. If in the textbook they say that there is a supremum but no infimum then they are not consistent.
      $endgroup$
      – drhab
      Dec 23 '18 at 9:41












    • $begingroup$
      Thus, as it is asserted that there exists a supremum, I may conclude that the textbook contradicts itself and is wrong.
      $endgroup$
      – Mew
      Dec 23 '18 at 9:44










    • $begingroup$
      I think so, yes.
      $endgroup$
      – drhab
      Dec 23 '18 at 9:44
















    4












    $begingroup$

    Looking at $S$ as a subset of $mathbb Q$ it has lower bounds, but has no infimum (i.e. greatest lower bound).



    Looking at $S$ as a subset of $mathbb R$ it does have an infimum which is $-sqrt2inmathbb R$.






    share|cite|improve this answer









    $endgroup$













    • $begingroup$
      Would the first case not mean there is also no supremum, though?
      $endgroup$
      – Mew
      Dec 23 '18 at 9:39












    • $begingroup$
      Indeed if $S$ is looked at as a subset of $mathbb Q$ (ordered by $leq$) it has no supremum either. If in the textbook they say that there is a supremum but no infimum then they are not consistent.
      $endgroup$
      – drhab
      Dec 23 '18 at 9:41












    • $begingroup$
      Thus, as it is asserted that there exists a supremum, I may conclude that the textbook contradicts itself and is wrong.
      $endgroup$
      – Mew
      Dec 23 '18 at 9:44










    • $begingroup$
      I think so, yes.
      $endgroup$
      – drhab
      Dec 23 '18 at 9:44














    4












    4








    4





    $begingroup$

    Looking at $S$ as a subset of $mathbb Q$ it has lower bounds, but has no infimum (i.e. greatest lower bound).



    Looking at $S$ as a subset of $mathbb R$ it does have an infimum which is $-sqrt2inmathbb R$.






    share|cite|improve this answer









    $endgroup$



    Looking at $S$ as a subset of $mathbb Q$ it has lower bounds, but has no infimum (i.e. greatest lower bound).



    Looking at $S$ as a subset of $mathbb R$ it does have an infimum which is $-sqrt2inmathbb R$.







    share|cite|improve this answer












    share|cite|improve this answer



    share|cite|improve this answer










    answered Dec 23 '18 at 9:38









    drhabdrhab

    102k545136




    102k545136












    • $begingroup$
      Would the first case not mean there is also no supremum, though?
      $endgroup$
      – Mew
      Dec 23 '18 at 9:39












    • $begingroup$
      Indeed if $S$ is looked at as a subset of $mathbb Q$ (ordered by $leq$) it has no supremum either. If in the textbook they say that there is a supremum but no infimum then they are not consistent.
      $endgroup$
      – drhab
      Dec 23 '18 at 9:41












    • $begingroup$
      Thus, as it is asserted that there exists a supremum, I may conclude that the textbook contradicts itself and is wrong.
      $endgroup$
      – Mew
      Dec 23 '18 at 9:44










    • $begingroup$
      I think so, yes.
      $endgroup$
      – drhab
      Dec 23 '18 at 9:44


















    • $begingroup$
      Would the first case not mean there is also no supremum, though?
      $endgroup$
      – Mew
      Dec 23 '18 at 9:39












    • $begingroup$
      Indeed if $S$ is looked at as a subset of $mathbb Q$ (ordered by $leq$) it has no supremum either. If in the textbook they say that there is a supremum but no infimum then they are not consistent.
      $endgroup$
      – drhab
      Dec 23 '18 at 9:41












    • $begingroup$
      Thus, as it is asserted that there exists a supremum, I may conclude that the textbook contradicts itself and is wrong.
      $endgroup$
      – Mew
      Dec 23 '18 at 9:44










    • $begingroup$
      I think so, yes.
      $endgroup$
      – drhab
      Dec 23 '18 at 9:44
















    $begingroup$
    Would the first case not mean there is also no supremum, though?
    $endgroup$
    – Mew
    Dec 23 '18 at 9:39






    $begingroup$
    Would the first case not mean there is also no supremum, though?
    $endgroup$
    – Mew
    Dec 23 '18 at 9:39














    $begingroup$
    Indeed if $S$ is looked at as a subset of $mathbb Q$ (ordered by $leq$) it has no supremum either. If in the textbook they say that there is a supremum but no infimum then they are not consistent.
    $endgroup$
    – drhab
    Dec 23 '18 at 9:41






    $begingroup$
    Indeed if $S$ is looked at as a subset of $mathbb Q$ (ordered by $leq$) it has no supremum either. If in the textbook they say that there is a supremum but no infimum then they are not consistent.
    $endgroup$
    – drhab
    Dec 23 '18 at 9:41














    $begingroup$
    Thus, as it is asserted that there exists a supremum, I may conclude that the textbook contradicts itself and is wrong.
    $endgroup$
    – Mew
    Dec 23 '18 at 9:44




    $begingroup$
    Thus, as it is asserted that there exists a supremum, I may conclude that the textbook contradicts itself and is wrong.
    $endgroup$
    – Mew
    Dec 23 '18 at 9:44












    $begingroup$
    I think so, yes.
    $endgroup$
    – drhab
    Dec 23 '18 at 9:44




    $begingroup$
    I think so, yes.
    $endgroup$
    – drhab
    Dec 23 '18 at 9:44











    2












    $begingroup$

    To me it looks like the textbook is wrong, and you have it right. The set does have lower bounds, and therefore a greatest lower bound (among the reals), and that's $-sqrt2$.






    share|cite|improve this answer









    $endgroup$


















      2












      $begingroup$

      To me it looks like the textbook is wrong, and you have it right. The set does have lower bounds, and therefore a greatest lower bound (among the reals), and that's $-sqrt2$.






      share|cite|improve this answer









      $endgroup$
















        2












        2








        2





        $begingroup$

        To me it looks like the textbook is wrong, and you have it right. The set does have lower bounds, and therefore a greatest lower bound (among the reals), and that's $-sqrt2$.






        share|cite|improve this answer









        $endgroup$



        To me it looks like the textbook is wrong, and you have it right. The set does have lower bounds, and therefore a greatest lower bound (among the reals), and that's $-sqrt2$.







        share|cite|improve this answer












        share|cite|improve this answer



        share|cite|improve this answer










        answered Dec 23 '18 at 9:36









        ArthurArthur

        117k7116200




        117k7116200






























            draft saved

            draft discarded




















































            Thanks for contributing an answer to Mathematics Stack Exchange!


            • Please be sure to answer the question. Provide details and share your research!

            But avoid



            • Asking for help, clarification, or responding to other answers.

            • Making statements based on opinion; back them up with references or personal experience.


            Use MathJax to format equations. MathJax reference.


            To learn more, see our tips on writing great answers.




            draft saved


            draft discarded














            StackExchange.ready(
            function () {
            StackExchange.openid.initPostLogin('.new-post-login', 'https%3a%2f%2fmath.stackexchange.com%2fquestions%2f3050189%2finfimum-of-the-set-x-in-mathbbq-x22%23new-answer', 'question_page');
            }
            );

            Post as a guest















            Required, but never shown





















































            Required, but never shown














            Required, but never shown












            Required, but never shown







            Required, but never shown

































            Required, but never shown














            Required, but never shown












            Required, but never shown







            Required, but never shown







            Popular posts from this blog

            To store a contact into the json file from server.js file using a class in NodeJS

            Redirect URL with Chrome Remote Debugging Android Devices

            Dieringhausen